a. 8
b. 4
c. 16
d. 12

A. 8b. 4c. 16d. 12

Answers

Answer 1

b. 4

....................

Answer 2

3x =24

or, x= 24/3

or, x =8


Related Questions

PLS HELP ASAP PLS PLSPLS
What is the explicit formula for this sequence?
6, 2, -2, -6, ...

Answers

Answer:

B

Step-by-step explanation:

this is just common sense. if you would have tried you could have easily answered this yourself.

the sequence starts with 6. that is a1.

so, the formula simply has to add "something" to 6.

therefore, A and D are out.

and then we see immediately that the sequence is built by subtracting 4 (or adding -4) for every step.

so, we need to add multiples of -4.

therefore, only B is correct (C adds multiples of +4).

Convert 13pi/6 to a degree measure

A=390
B=2450.44
C=30
D=780

Answers

Answer:

390 degrees

Step-by-step explanation:

The conversion factor is 180/pi

13 pi /6 * 180/pi

13/6 *180

390

Find x.

A. √-2
B. 4
C. √2/2
D. √6/2

Answers

Answer:

sqrt(2)

Step-by-step explanation:

Since this is a right triangle, we can use trig functions

sin theta = opp / hyp

sin 45 = x/2

2 sin 45 =x

2 (sqrt(2)/2) =x

sqrt(2) =x

An exponential function fx) is reflected across the y-axis to create functiong(x). Which is a true statement
regarding fa) and g(x)?
The two functions have no points in common
The two functions have the same initial value
The two function have opposite output values of each other for any given input value
The graph of the two functions would look exactly the same
Intro

Answers

Answer:

The two functions have the same initial value

can someone please help me solve this?

Answers

Answer:

-15

Step-by-step explanation:

df(x, y) / dy = lim (f(x, y+h) - f(x, y)) / h =

lim (e^x^2 - 15y - 15h - e^x^2 ‐ 15y) / h = lim -15h / h = -15

so, the first derivative of f(x,y) by y is simply a constant : -15

fy (x, y) = -15 for any and every values of x and y.

the fast track would be derivative calculation :

the derivative by y makes x a constant. and every constant is eliminated for derivation.

so, e^x^2 goes away (= 0 in the derivative).

that leaves -15y = -15×y¹. its derivative simply is

1×-15×y⁰ = -15

What is the volume of the cone to the nearest whole number?
20,520pi cm3
43,320pi cm3
86,640pi cm3
136,094pi cm3

Answers

C. 86,640pi cm3


Hope this helps

[tex] \large\begin{gathered} {\underline{\boxed{ \rm {\red{Volume \: of \: Cone \: = \: \pi \: {r}^{2} \: \frac{h}{3} }}}}}\end{gathered}[/tex]

r represents radius of cone

h represents height of cone

height of cone is 90 cm

radius of cone is diameter / 2

‌ ⇛76/2 = 38

‌ ⇛radius = 38 cm

Solution

[tex]\bf \Large \longrightarrow \:π \: × \: (38 \: cm)² \: × \: \frac{90 }{3} [/tex]

[tex]\bf \Large \longrightarrow \: \pi \: \times \: \:1444 \: {cm}^{2} \: \times \: \cancel\frac{90}{3} \: \: ^{30 \: cm} [/tex]

⇥ 43,320 π cm³

Option ( B ) is the correct answer

I
7. The total cost to rent a truck is $100 and $0.20 per km.
a. Determine an algebraic model for the relationship between total cost and distance driven. Use C to
represent total cost (S) to rent the truck and d to represent distance driven (km).

Answers

Answer:

C= 100+ d*0.2

Step-by-step explanation:

Give data

Cost of rental = $100

Cost per km= $0.2

Total cost= C

Distance= d

Let us model the expression

Hence the expression is given as

C= 100+ d*0.2

PLS HELP ME ASAP!!!!

Answers

The y row changes by 3 each spot. The x row changed by 4 each spot. So the slope is 3/4x.

simplify the following​

Answers

Answer:

1) 11[tex]\sqrt{3}[/tex]

2) 2[tex]\sqrt{2}[/tex]

3) [tex]20\sqrt{3} + 15\sqrt{2}[/tex]

4) [tex]53 + 12\sqrt{10}[/tex]

5) -2

6) [tex]7\sqrt{2} - 5\sqrt{3}[/tex]

Step-by-step explanation:

1) 2[tex]\sqrt{12}[/tex] + 3[tex]\sqrt{48}[/tex] - [tex]\sqrt{75}[/tex]

=(2 × 2[tex]\sqrt{3}[/tex] )+ (3 × 4[tex]\sqrt{3}[/tex]) - 5[tex]\sqrt{3}[/tex]

= 4[tex]\sqrt{3}[/tex] + 12[tex]\sqrt{3}[/tex] - 5[tex]\sqrt{3}[/tex]

= 11[tex]\sqrt{3}[/tex]

2) 4[tex]\sqrt{8}[/tex] -2[tex]\sqrt{98}[/tex] + [tex]\sqrt{128}[/tex]

= (4 × 2[tex]\sqrt{2}[/tex]) - (2 × 7[tex]\sqrt{2}[/tex]) + 8[tex]\sqrt{2}[/tex]

= 8[tex]\sqrt{2}[/tex] - 14[tex]\sqrt{2}[/tex] +8[tex]\sqrt{2}[/tex]

= 2[tex]\sqrt{2}[/tex]

3) 5[tex]\sqrt{12\\}[/tex] - 3[tex]\sqrt{18} + 4 \sqrt{72} +2\sqrt{75}[/tex]

= 5× [tex]2\sqrt{3}[/tex] - 3×[tex]3\sqrt{2}[/tex] + 4×[tex]6\sqrt{2}[/tex] + 2×[tex]5\sqrt{3}[/tex]

= [tex]10\sqrt{3} - 9\sqrt{2} +24\sqrt{2} +10\sqrt{3}[/tex]

= [tex]20\sqrt{3} + 15\sqrt{2}[/tex]

4) [tex](2\sqrt{2} + 3\sqrt{5} )^{2}[/tex]

= [tex]8 + 12\sqrt{10} + 45[/tex]

= [tex]53 + 12\sqrt{10}[/tex]

5) [tex](1+\sqrt{3} ) (1-\sqrt{3} )[/tex]

= [tex]1 - 3[/tex]

= -2

6) [tex](2\sqrt{6} -1) (\sqrt{3} -\sqrt{2} )[/tex]

= [tex]2\sqrt{18}-2\sqrt{12} -\sqrt{3} +\sqrt{2}[/tex]

= 2×[tex]3\sqrt{2}[/tex] - 2×[tex]2\sqrt{3}[/tex] - [tex]\sqrt{3} + \sqrt{2}[/tex]

= [tex]6\sqrt{2} - 4\sqrt{3} -\sqrt{3} +\sqrt{2}[/tex]

= [tex]7\sqrt{2} - 5\sqrt{3}[/tex]

Hope the working out is clear and will help you. :)

Step-by-step explanation:

please I solved the question in the diagram above

5^-4 over 5^3
simplify
A. 5^7
B. 5^-1
C. 1/5
D. 1/5^7​

Answers

Answer:

D is the correct answer.

rational or irrational? and why

Answers

Answer:

Irrational

Step-by-step explanation:

If we evaluate the square root, we realize that the number will not be a perfect square.

Since it is not a perfect square, it will be an unending, never-repeating, decimal.

Therefore, it will be irrational.

What is equivalent to 1/6

Answers

Answer:

0.16666...

6/36

2/12

3/18

4/24

5/30

There's a lot of things.

2cos5xcos3x+sinx=cos8x

Answers

It looks like your equation (it's not an identity) is

2 cos(5x) cos(3x) + sin(x) = cos(8x)

Recall that

cos(x + y) = cos(x) cos(y) - sin(x) sin(y)

cos(x - y) = cos(x) cos(y) + sin(x) sin(y)

==>   2 cos(x) cos(y) = cos(x + y) + cos(x - y)

so that

2 cos(5x) cos(3x) = cos(8x) + cos(2x)

Then the equation simplifies to

cos(8x) + cos(2x) + sin(x) = cos(8x)

cos(2x) + sin(x) = 0

Also recall that

cos(2x) = 1 - 2 sin²(x)

so the equation is quadratic in sin(x) and can be factorized:

1 - 2 sin²(x) + sin(x) = 0

2 sin²(x) - sin(x) - 1 = 0

(2 sin(x) + 1) (sin(x) - 1) = 0

Solve for x :

2 sin(x) + 1 = 0   or   sin(x) - 1 = 0

sin(x) = -1/2   or   sin(x) = 1

[x = arcsin(-1/2) + 2   or   x = π - arcsin(-1/2) + 2]   or   x = arcsin(1) + 2

(where n is any integer)

x = -π/6 + 2   or   x = -5π/6 + 2   or   x = π/2 + 2

a grocery store buys boxes of cereal for $ 2.00 each and sells them for 50% more. what does the grocery store charge its customers for each box of cereal?

Answers

Answer:

$3.00

Step-by-step explanation:

First find the markup

2.00 * 50%

2 * .5

1

Add this to the original price

2+1 =3

The store sells the cereal for $3.00

Answer:

$3

Step-by-step explanation:

The grocery store pays $2.00 for each box of cereal bought.

First, find 50% of the cost of each box:

2.00 x 0.50 = 1.00

Next, add the additional amount to the starting price:

2.00 + 1.00 = $3.00

$3.00 is your answer.

~

Solve for x: 5 [(x/6)-2] = 2x + 4

Answers

Answer:

-12 =x

Step-by-step explanation:

5 [(x/6)-2] = 2x + 4

Distribute

5/6x - 10 = 2x+4

Subtract 5/6x from each side

5/6x -10 -5/6x = 2x -5/6x +4

-10 =2x -5/6x +4

Subtract 4 from each side

-10-4 = 2x - 5/6x

Get a common denominator for the right side

-14 = 12/6x - 5/6x

-14 = 7/6x

Multiply each side by 6/7

-14 * 6/7 = 7/6 x * 6/7

-12 =x

Answer:

x = -12

Step-by-step explanation

The number seventeen and seven thousand in decimal form is​

Answers

Answer:

17.00 and 17,000.00

this is your answer to ur question

The blue team scored two more than five times the number of points,p, scored by the red team
Write an expression for the problem

Answers

Answer:

2 + 5p

Step-by-step explanation:

Hi there!

Let [tex]p[/tex] equal the number of points scored by the red team.

We're given:

The blue team scored two more than five times the number of points scored by the red team.

⇒ blue team points = 2 + (5 × red team points)

⇒ blue team points = 2 + 5p

I hope this helps!

(-9) + (-4) =
(+3) - (-5) =
(+2)x(+2) =
(+36) + (+4)
(+5) X (+7) =
(+8) (+9) =
(-4) + (-2) =
(+3) -(-6) =
(-6)*(-3) =
(-4) + (-4)
(+5) X (+7)
(+4)-(-5) =
(+4) - (+3) =
(+10) + (+5)
(+3) (+8)

Answers

[tex]\\ \bull\sf\longmapsto -9+(-4)=-9-4=-13[/tex]

[tex]\\ \bull\sf\longmapsto +3-(-5)=3+5=8[/tex]

[tex]\\ \bull\sf\longmapsto 2\times 2=4[/tex]

[tex]\\ \bull\sf\longmapsto 36+4=40[/tex]

[tex]\\ \bull\sf\longmapsto 5\times 7=35[/tex]

[tex]\\ \bull\sf\longmapsto 8(9)=72[/tex]

[tex]\\ \bull\sf\longmapsto -4+(-2)=-4-2=-6[/tex]

[tex]\\ \bull\sf\longmapsto 3-(-6)=3+6=9[/tex]

[tex]\\ \bull\sf\longmapsto -6(-3)=18[/tex]

[tex]\\ \bull\sf\longmapsto -4+(-4)=-4-4=-8[/tex]

[tex]\\ \bull\sf\longmapsto 5(7)=35[/tex]

[tex]\\ \bull\sf\longmapsto 4-(-5)=4+5=9[/tex]

[tex]\\ \bull\sf\longmapsto 4-3=1[/tex]

[tex]\\ \bull\sf\longmapsto 10+5=15[/tex]

[tex]\\ \bull\sf\longmapsto 3(8)=24[/tex]

Identify the volume and surface area of the hemisphere in terms of pie.

Answers

Answer:

it's off topic but which grade are you in

20 is 15% of what number?
PLEASE ANSWER ILL GIVE BRAINLIESTTTT

Answers

Answer:

133.33

Step-by-step explanation:

Answer:

133 1/3 or 133.33

Step-by-step explanation:

So first let's turn 15% into a decimal. For me personally, its less intimidating that way.

15%=0.15

now, let's say the number we are trying to find is x.

A trick I use for these questions is to look at the words. "Of" is used when something is multiplied. "Is" is used for equal signs. So if 20 is 0.15 of x, our formula is easy to figure out!

20=0.15x

x=133.33333...

x=133 1/3

Simplify 2m^2 – 2m + 3m^2

Answers

Answer:

5m^2-2m

Step-by-step explanation:

2m^2-2m + 3m^2

5m^2-2m

Answer:

5m² - 2m

Step-by-step explanation:

Given

2m² - 2m + 3m² ← collect like terms

= (2m² + 3m²) - 2m

= 5m² - 2m

Use the graph to find the constant of proportionality (unit rate)

Answers

Answer:

9) 10

10) 5

Step-by-step explanation:

Please help explanation if possible

Answers

Answer:

(1, - 2 )

Step-by-step explanation:

Given the equations

3x - 4y = 11 → (1)

y + 3x = 1 ( subtract 3x from both sides )

y = 1 - 3x → (2)

Substitute y = 1 - 3x into (1)

3x - 4(1 - 3x) = 11 ( distribute and simplify left side )

3x - 4 + 12x = 11

15x - 4 = 11 ( add 4 to both sides )

15x = 15 ( divide both sides by 15 )

x = 1

Substitute x = 1 into (2) for corresponding value of y

y = 1 - 3(1) = 1 - 3 = - 2

solution is (1, - 2 )

Let x be the sum of the following 2000 numbers​

Answers

Answer:

See attached image for the sum computed using wolfram alpha, raw value is: 49382716049382716049382716049382716049382716049382716049382716049382716049382716049382716049382716049382716049382716049382716049382716049382716049382716049382716049382716049382716049382716049382716049382716049382716049382716049382716049382716049382716049382716049382716049382716049382716049382716049382716049382716049382716049382716049382716049382716049382716049382716049382716049382716049382716049382716049382716049382716049382716049382716049382716049382716049382716049382716049382716049382716049382716049382716049382716049382716049382716049382716049382716049382716049382716049382716049382716049382716049382716049382716049382716049382716049382716049382716049382716049382716049382716049382716049382716049382716049382716049382716049382716049382716049382716049382716049382716049382716049382716049382716049382716049382716049382716049382716049382716049382716049382716049382716049382716049382716049382716049382716049382716049382716049382716049382716049382716049382716049382716049382716049382716049382716049382716049382716049382716049382716049382716049382716049382716049382716049382716049382716049382716049382716049382716049382716049382716049382716049382716049382716049382716049382716049382716049382716049382716049382716049382716049382716049382716049382716049382716049382716049382716049382716049382716049382716049382716049382716049382716049382716049382716049382716049382716049382716049382716049382716049382716049382716049382716049382716049382716049382716049382716049382716049382716049382716049382716049382716049382716049382716049382716049382716049382716049382716049382716049382716049382716049382716049382716049382716049382716049382716049382716049382716049382716049382716049382716049382716049382716049382716049382716049382716049382716049382716049382716049382716049382716049382716049382716049382716049382716049382716049382716049382716049382716049382716049382716049382716049382716049382716049382716049382716049382716049382716049382716049382716049382716049382716049382716049382716049382716049382716049382715160

Answer:

Step-by-step explanation:

First, i divide all terms by 4

sequence is 1,11,111,1111,....,1111...1111 (with 2000 digits)

[tex]u_1=1\\\\u_2=11=10+1=\dfrac{10^2-1}{9}\\\\u_3=111=100+10+1=\dfrac{10^3-1}{9}\\\\u_4=1111=1000+100+10+1=\dfrac{10^4-1}{9}\\....\\u_n=\dfrac{10^n-1}{9}\\\\\\\displaystyle \sum_{i=1}^n\ u_1 =\dfrac{10^1-1}{9}+\dfrac{10^2-1}{9}+\dfrac{10^3-1}{9}+...+\dfrac{10^n-1}{9}\\=\dfrac{1}{9} *(10+10^2+10^3+....+10^n+1-1-(1+1+1+1...+1)\ )\\\\=\dfrac{1}{9}*( \frac{10^{n+1}-1}{9} -(n+1)\ )[/tex]

[tex]\boxed{4+44+444+....+444...4=\dfrac{4}{9}*( \dfrac{10^{2001}-1}{9} -2001\ )}\\\\\\\approx{4,9382716049382716049382716049383*10^{1999} }[/tex]

Which correlation best describes the data below.

Answers

Answer:

Weak negative correlation

The number of pounds of one-dollar-a-pound
coffee needed to mix with 80 pounds of 70¢ a
pound coffee to make a mixture worth 84¢ a
pound is
(A) 70
(B) 80
(C) 95
(D) 65

Answers

Answer:

A

Step-by-step explanation:

Let's say we need x pounds of one-dollar-a-pound coffee . The coffee must average out to 84 cents a pound, and the formula for average is

sum of cost of coffee / number of pounds of coffee, so we have

0.84 = total cost of coffee / (x+80) . The total cost of coffee can be found to be the sum of the cost of $1 coffee and 70 cent coffee, so we have

0.84 = (cost of $1 coffee + cost of 70 cent coffee) / (x+80)

The cost of $1 coffee can be found by adding $1 for each pound of one dollar coffee, or $1 * x. Similarly, the cost of 70 cent coffee is equal to 0.70 * 80, so we have

0.84 = (1*x+0.7*80)/(x+80)

0.84 = (x+56)/(x+80)

multiply both sides by (x+80) to remove a denominator

0.84(x+80) = x+56

0.84x + 67.2 = x+56

subtract both sides by 56 and 0.84x to isolate the x and its coefficients

11.2 = 0.16 x

divide both sides by 0.16 to isolate x

11.2/0.16 = x = 70

The number of pounds of a constituent in a mixture given the cost of the

mixture and the cost and mass of the other constituent can be calculated

by using an equation to model the system

The correct option for the number of pounds of one-dollar- pound coffee needed is option A

(A) 70 pounds

The procedure for arriving at the correct option is as follows:

The given parameters are;

The cost of the the coffee for which the mass in the mixture is to be determined = One-Dollar a pound = 100 ¢ a pound

The mass of the coffee 70¢ a pound coffee to be mixed = 80 pounds

The cost per pound of the mixture = 84 ¢ a pound

The required parameter;

The number of pounds of the one-dollar-a-pound (100 ¢ a pound) coffee in the mixture

Method:

Let x (pound) represent the number of pounds of the one-dollar-a-pound coffee in the mixture, we have;

Mass of mixture = Mass of the one-dollar-a-pound in the mixture, x + Mass of 70 ¢ a pound in the mixture, 80

∴ Mass of mixture in pounds = x + 80

Cost = Cost per pound × Number of pound

Find solution by applying the equation;

Cost of the constituents = Cost of the mixture

Where;

Cost of the constituents = $1 × x + 70 ¢ × 80 = 100 ¢ × x  + 70 ¢ × 80

Cost of the mixture = 84 ¢ × (x + 80)

Therefore;

100 ¢ × x  + 70 ¢ × 80 = 84 ¢ × (x + 80)

The above can be expressed as 100·x + 70×80 = 84 × (x + 80)

Expanding, evaluating and collecting like terms gives;

100·x + 5,600 = 84·x + 6,720

100·x - 84·x = 6,720 - 5,600 = 1,120

16·x = 1,120

x = 1,120/16 = 70

The number of pounds of one-dollar- pound coffee needed, x = 70 pounds

Learn more about equation modelling here;

https://brainly.com/question/14102741

Write an equation of the line. What is the equation of the line?

Answers

Answer:

y=-1

Step-by-step explanation:

As the line is horizontal in nature and pass through (2,-1), the equation is y=-1

Find the measure of one interior angle for the following regular polygon

Answers

To find the sum of the interior angles you use the formula (n-2)(180), where n is the number of sides. In this case, the number of sides is 12, so you get 1800 degrees. Then you divide by number of angles which is 12. So your answer is 150 degrees.

What is the 6th term of this pattern : 0.1, 0.02, 0.12, 0.14, 0.26, ____?




If you will solve i will give brainliest.

Answers

Answer:

0.28

Step-by-step explanation:

Can someone help me out

Answers

It equals 5? The markings means it’s equal right?
Other Questions
b) Use Greens theorem to findx^2 ydx-xy^2 dy where C is the circle x2 + y2 = 4 going counter clock wise. PEASE HELP. What are the two main sources of energy on earth? Explain some things that each source creates/powers/ does for earth. Read the passage.Clean & Green laundry detergent removes stains better than other leading laundry detergents. But Clean & Green is safe for the environment, too. So buy Clean & Green. Your clothes get clean, and the environment stays green.Which statement about this advertisement is true? Which of the following stores the operating system?O Von Neumann centerO Bluetooth centerO the cloudO random access memory find the missing side corruption is a social crime.why? The nucleus in a body cell of a fly contains 12 chromosomes.How many chromosomes are there in the nucleus of a sperm from this fly?a) 3 b) 6 c) 12 d) 24 Salary paid to Saroj by cheque Rs. 35,000 trun in negativea..She reads a book. Which equation can be used to find the length of Line segment A C? 8/3=12/n solve for n Assume that the matrices below are partitioned conformably for block multiplication. Compute the product. [I 0] [W X][K I] [Y Z] Meaning of power in physics pls answer and you will be blessed :) JacksonIndustries produces two products. The products' estimated costs are as follows: Product A Product BDirect Materials $20,000 $15,000Direct Labor $30,000 $10,000 The company's overhead costs of $200,000 are allocated based on labor cost. Assume 4,000 units of product A and 5,000 units of Product B are produced. What is the total amount of production costs that would be assigned to Product A? (Do not round intermediate calculations.)a. $200,000b. $75,000c. $50,000 d .$150,000 e. $114,285.71 Show what happens to the firm's output choice and profit if the price of the product falls from $52 to $42. If the market price falls from $52 to $42, then the firm's output will decrease or increase from _____units to _____ units. (Enter your responses using integers.) PLS HELP UNSCRAMBLE THE WORDS IN SPANISH what does "form a more perfect union " mean A history teacher gives a 17 question True or false exam. In how many different ways can the test be answered if the possible answers are true or false or possibly to leave the answer blank? Suppose Event A is taking 15 or more minutes to get to work tomorrow and Event B is taking less than 15 minutes to get to work tomorrow. Events A and B are said to be complementary events.a. Trueb. False